§1. Bất đẳng thức

Lục Hoàng Phong

Cho x, y, z > 0 thỏa mãn \(x^2+y^2+z^2\ge1\)

Chứng minh: \(\dfrac{x^3}{y}+\dfrac{y^3}{z}+\dfrac{z^3}{x}\ge1\)

Mong mọi người giúp ạ..........Em sẽ đội ơn cả đời

Nguyễn Huy Tú
1 tháng 8 2017 lúc 20:39

Áp dụng bất đẳng thức Cauchy-Shwarz dạng Engel và \(a^2+b^2+c^2\ge ab+bc+ca\) có:
\(\dfrac{x^3}{y}+\dfrac{y^3}{z}+\dfrac{z^3}{x}=\dfrac{x^4}{xy}+\dfrac{y^4}{yz}+\dfrac{z^4}{xz}\ge\dfrac{\left(x^2+y^2+z^2\right)^2}{xy+yz+xz}\)

\(\ge\dfrac{\left(x^2+y^2+z^2\right)^2}{x^2+y^2+z^2}=x^2+y^2+z^2\ge1\)

Dấu " = " khi x = y = z = \(\dfrac{1}{\sqrt{3}}\)

Vậy...

Bình luận (5)
Serena chuchoe
1 tháng 8 2017 lúc 20:49

Cách khác nhé!/-/

Áp dụng BĐT Holder ta có:

\(\left(\dfrac{x^3}{y}+\dfrac{y^3}{z}+\dfrac{z^3}{x}\right)\left(\dfrac{x^3}{y}+\dfrac{y^3}{z}+\dfrac{z^3}{x}\right)\left(y^2+z^2+x^2\right)\ge\left(x^2+y^2+z^2\right)^3\)

Do đó \(\dfrac{x^3}{y}+\dfrac{y^3}{z}+\dfrac{z^3}{x}\ge x^2+y^2+z^2\ge1\)

Đẳng thức xảy ra khi \(x=y=z=\dfrac{1}{\sqrt{3}}\)

Bình luận (1)
Đào Ngọc Hoa
1 tháng 8 2017 lúc 20:57

Ta có: \(x^2+y^2+z^2\ge3\sqrt[3]{x^2y^2z^2}\)

\(\Rightarrow3\sqrt[3]{x^2y^2z^2}\ge1\)

lại có: \(\dfrac{x^3}{y}+\dfrac{y^3}{z}+\dfrac{z^3}{x}\ge3.\sqrt[3]{x^2y^2z^2}\)

\(\Rightarrow\dfrac{x^3}{y}+\dfrac{y^3}{z}+\dfrac{z^3}{x}\ge1\)

Bình luận (1)

Các câu hỏi tương tự
Mẫn Đan
Xem chi tiết
Dương Nhật Hoàng
Xem chi tiết
ngân hồng
Xem chi tiết
Phan Cả Phát
Xem chi tiết
Trần Minh Tâm
Xem chi tiết
Nguyễn Thanh
Xem chi tiết
Phan Cả Phát
Xem chi tiết
Nguyễn Thanh
Xem chi tiết
Phạm Kim Oanh
Xem chi tiết